LSAT and Law School Admissions Forum

Get expert LSAT preparation and law school admissions advice from PowerScore Test Preparation.

User avatar
 Dave Killoran
PowerScore Staff
  • PowerScore Staff
  • Posts: 5853
  • Joined: Mar 25, 2011
|
#59496
Complete Question Explanation
(The complete setup for this game can be found here: lsat/viewtopic.php?t=26462)

The correct answer choice is (A)


There are two ways to answer this question. The first way is to use the rules to eliminate each of the incorrect answer choices. The second is to use the templates to see which player could possibly play in two consecutive games. Let’s examine both approaches:


Use the Rules

This is the easiest and fastest approach. The third and fourth rules tell us that N and O play in exactly one game, and so answer choices (B) and (C) can be eliminated. The fourth rule also indicates that while S plays in two games, those two games cannot be consecutive, so answer choice (D) can be eliminated. The first rule states that T cannot play in the first or third games, and so T could not play in consecutive games, and answer choice (E) can be eliminated. Thus, answer choice (A) is proven correct by process of elimination.

Use the Templates

When examining the templates, always look at Template #2 first because it contains more specific information. In Template #2, none of the variables could play in consecutive games, so we automatically know that the variable that plays consecutively does so under the parameters of Template #1.

Template #1 immediately eliminates all players except L, P, or T, because only L, P, or T could play twice and could conceivably be consecutive (although S plays twice, the template shows that S is not consecutive and is thus eliminated). However, P is not one of the answer choices, so the answer must be L or T. Of course, as shown on the template, T cannot be consecutive, and so L is the player who could play consecutively and answer choice (A) is the correct answer.


Answer choice (A): This is the correct answer choice.

Answer choices (B), (C), (D), and (E): The players in each of these answer cannot play in consecutive games, and thus each one of these answers is incorrect.

Get the most out of your LSAT Prep Plus subscription.

Analyze and track your performance with our Testing and Analytics Package.